Spin vs orbital angular momenta in QFTAddition of Angular Momementa in deeply bound situations, proton spin...

What is the meaning of "you has the wind of me"?

How were the LM astronauts supported during the moon landing and ascent? What were the max G's on them during these phases?

How to get the two pictures aligned

Reduce column width of table while also aligning values at decimal point

How to contact Apple to check if they will permit an iOS app?

Do the licences permit GPL- and BSD-licensed applications to be used for government work?

Inadvertently nuked my disk permission structure - why?

Invert Some Switches on a Switchboard

Explanation for a joke about a three-legged dog that walks into a bar

Why are there not any MRI machines available in Interstellar?

What should I say when a company asks you why someone (a friend) who was fired left?

Iterate over non-const variables in C++

USA: Can a witness take the 5th to avoid perjury?

Can two figures have the same area, perimeter, and same number of segments have different shape?

What to do when you reach a conclusion and find out later on that someone else already did?

How do I stop my characters falling in love?

Time travel novel: machine makes clones, clones battle to be the one to get back their life

How to write a sincerely religious protagonist without preaching or affirming or judging their worldview?

Is my employer paying me fairly? Going from 1099 to W2

Send a single HTML email from Thunderbird, overriding the default "plain text" setting

How do campaign rallies gain candidates votes?

What does "see" in "the Holy See" mean?

How can I receive packages while in France?

Difficulty pronouncing "maths", "baths", "hundredths", "sixths"



Spin vs orbital angular momenta in QFT


Addition of Angular Momementa in deeply bound situations, proton spin crisisHow does spin appear in QFT?Total angular momentum in QFTDoes orbital angular mometum has no meaning for single photons?Spin Orbital Coupling matrix in p-orbital basisSpin operators in QMSpin and orbital angular momentum parity transformation in 2DHow is the product $Lcdot S$ between orbital and spin angular momentum operators defined? Do they act on the same or different Hilbert spaces?Decay of spin-1 particle into two spin-0 particlesAngular momentum coupling






.everyoneloves__top-leaderboard:empty,.everyoneloves__mid-leaderboard:empty,.everyoneloves__bot-mid-leaderboard:empty{ margin-bottom:0;
}







6












$begingroup$


I couldn't find past answers that quite match what I wanted, so I will try to ask in slightly different manner. In QM, we have total angular momentum operator $vec{J}$ (I dropped the hat for convenience)
begin{align}
vec{J} = vec{L} +vec{S}
end{align}

where $vec{L}$ is for orbital angular momentum and $vec{S}$ is for spin angular momentum. For hydrogen atom, this (not addition, but rather $vec{L}^2$ and $L_z$) gives us two of three quantum numbers, thus we often label the state for fixed energy as $left|l,mright>$ (for some reason $m$ is used in standard texts instead of, say, $s$). I think I understand how they work in QM, or at least the fact that $vec{J},vec{L},vec{S}$ all share the same commutation relations (i.e. share the same Lie algebra structure). The addition comes from when one has stuffs like spin-orbit coupling and the like.



In QFT, stuff starts to get a bit/lot confusing. First, angular momentum operator is really used to label states in the irreducible unitary representation of the Poincare group, which is written as $left|p,sigmaright>$ (or $Psi_{p,s}$ following Weinberg); it is an operator in the sense that it appears as a representation of rotation element of the group. Wigner's classification then gives us that we also label states with two quantum numbers: we sometimes also write this as $left|m,jright>$, where $m$ is the rest mass and $j$ is the spin. Nowhere in any of these constructions do I see the conventional orbital angular momentum (in the text by Schwartz Chapter 11, for example, he made a comment when there is no angular momentum, so he got the "easy" case of $vec{J}=vec{S}$.) In standard QM, we clearly distinguish two versions of angular momenta and their eigenvalues.



Question: does orbital angular momentum make sense in QFT and how does it arise if it does? Should I think of $vec L$ as furnishing some kind of tensor product representation so that I should be labelling states with $left|m,j,sright>$ or something? Another possibility is that one simply does not work with addition of angular momenta in QFT, especially in free theory, but I would like to have an explanation on why this should or should not be the case. I may have understood something really basic but I can't fish out exactly what.



Note: I believe the usual thing about spin-orbit coupling should not work, because that's QM (think about hydrogen atom) and strictly speaking QFT, unless we do something more (2-particle states?). In those cases, the operator $vec{L}$ even comes from $vec{r}timesvec{p}$ which does not appear naturally in QFT (what's $vec{r}$ in QFT?). I don't think I should go and define $vec{L}:=vec{r}timesvec{hat{pi}}$, where $hatpi$ is the conjugate momentum of the field.










share|cite|improve this question











$endgroup$












  • $begingroup$
    Why is "that not QFT?" Angular momentum, both spin and orbital, is strongly related to rotational symmetry. (Paging E. Noether. Will E. Noether please pick up a white house phone.) You still have symmetry in QFT. You still get the same reps of the rotation group.
    $endgroup$
    – puppetsock
    8 hours ago


















6












$begingroup$


I couldn't find past answers that quite match what I wanted, so I will try to ask in slightly different manner. In QM, we have total angular momentum operator $vec{J}$ (I dropped the hat for convenience)
begin{align}
vec{J} = vec{L} +vec{S}
end{align}

where $vec{L}$ is for orbital angular momentum and $vec{S}$ is for spin angular momentum. For hydrogen atom, this (not addition, but rather $vec{L}^2$ and $L_z$) gives us two of three quantum numbers, thus we often label the state for fixed energy as $left|l,mright>$ (for some reason $m$ is used in standard texts instead of, say, $s$). I think I understand how they work in QM, or at least the fact that $vec{J},vec{L},vec{S}$ all share the same commutation relations (i.e. share the same Lie algebra structure). The addition comes from when one has stuffs like spin-orbit coupling and the like.



In QFT, stuff starts to get a bit/lot confusing. First, angular momentum operator is really used to label states in the irreducible unitary representation of the Poincare group, which is written as $left|p,sigmaright>$ (or $Psi_{p,s}$ following Weinberg); it is an operator in the sense that it appears as a representation of rotation element of the group. Wigner's classification then gives us that we also label states with two quantum numbers: we sometimes also write this as $left|m,jright>$, where $m$ is the rest mass and $j$ is the spin. Nowhere in any of these constructions do I see the conventional orbital angular momentum (in the text by Schwartz Chapter 11, for example, he made a comment when there is no angular momentum, so he got the "easy" case of $vec{J}=vec{S}$.) In standard QM, we clearly distinguish two versions of angular momenta and their eigenvalues.



Question: does orbital angular momentum make sense in QFT and how does it arise if it does? Should I think of $vec L$ as furnishing some kind of tensor product representation so that I should be labelling states with $left|m,j,sright>$ or something? Another possibility is that one simply does not work with addition of angular momenta in QFT, especially in free theory, but I would like to have an explanation on why this should or should not be the case. I may have understood something really basic but I can't fish out exactly what.



Note: I believe the usual thing about spin-orbit coupling should not work, because that's QM (think about hydrogen atom) and strictly speaking QFT, unless we do something more (2-particle states?). In those cases, the operator $vec{L}$ even comes from $vec{r}timesvec{p}$ which does not appear naturally in QFT (what's $vec{r}$ in QFT?). I don't think I should go and define $vec{L}:=vec{r}timesvec{hat{pi}}$, where $hatpi$ is the conjugate momentum of the field.










share|cite|improve this question











$endgroup$












  • $begingroup$
    Why is "that not QFT?" Angular momentum, both spin and orbital, is strongly related to rotational symmetry. (Paging E. Noether. Will E. Noether please pick up a white house phone.) You still have symmetry in QFT. You still get the same reps of the rotation group.
    $endgroup$
    – puppetsock
    8 hours ago














6












6








6


4



$begingroup$


I couldn't find past answers that quite match what I wanted, so I will try to ask in slightly different manner. In QM, we have total angular momentum operator $vec{J}$ (I dropped the hat for convenience)
begin{align}
vec{J} = vec{L} +vec{S}
end{align}

where $vec{L}$ is for orbital angular momentum and $vec{S}$ is for spin angular momentum. For hydrogen atom, this (not addition, but rather $vec{L}^2$ and $L_z$) gives us two of three quantum numbers, thus we often label the state for fixed energy as $left|l,mright>$ (for some reason $m$ is used in standard texts instead of, say, $s$). I think I understand how they work in QM, or at least the fact that $vec{J},vec{L},vec{S}$ all share the same commutation relations (i.e. share the same Lie algebra structure). The addition comes from when one has stuffs like spin-orbit coupling and the like.



In QFT, stuff starts to get a bit/lot confusing. First, angular momentum operator is really used to label states in the irreducible unitary representation of the Poincare group, which is written as $left|p,sigmaright>$ (or $Psi_{p,s}$ following Weinberg); it is an operator in the sense that it appears as a representation of rotation element of the group. Wigner's classification then gives us that we also label states with two quantum numbers: we sometimes also write this as $left|m,jright>$, where $m$ is the rest mass and $j$ is the spin. Nowhere in any of these constructions do I see the conventional orbital angular momentum (in the text by Schwartz Chapter 11, for example, he made a comment when there is no angular momentum, so he got the "easy" case of $vec{J}=vec{S}$.) In standard QM, we clearly distinguish two versions of angular momenta and their eigenvalues.



Question: does orbital angular momentum make sense in QFT and how does it arise if it does? Should I think of $vec L$ as furnishing some kind of tensor product representation so that I should be labelling states with $left|m,j,sright>$ or something? Another possibility is that one simply does not work with addition of angular momenta in QFT, especially in free theory, but I would like to have an explanation on why this should or should not be the case. I may have understood something really basic but I can't fish out exactly what.



Note: I believe the usual thing about spin-orbit coupling should not work, because that's QM (think about hydrogen atom) and strictly speaking QFT, unless we do something more (2-particle states?). In those cases, the operator $vec{L}$ even comes from $vec{r}timesvec{p}$ which does not appear naturally in QFT (what's $vec{r}$ in QFT?). I don't think I should go and define $vec{L}:=vec{r}timesvec{hat{pi}}$, where $hatpi$ is the conjugate momentum of the field.










share|cite|improve this question











$endgroup$




I couldn't find past answers that quite match what I wanted, so I will try to ask in slightly different manner. In QM, we have total angular momentum operator $vec{J}$ (I dropped the hat for convenience)
begin{align}
vec{J} = vec{L} +vec{S}
end{align}

where $vec{L}$ is for orbital angular momentum and $vec{S}$ is for spin angular momentum. For hydrogen atom, this (not addition, but rather $vec{L}^2$ and $L_z$) gives us two of three quantum numbers, thus we often label the state for fixed energy as $left|l,mright>$ (for some reason $m$ is used in standard texts instead of, say, $s$). I think I understand how they work in QM, or at least the fact that $vec{J},vec{L},vec{S}$ all share the same commutation relations (i.e. share the same Lie algebra structure). The addition comes from when one has stuffs like spin-orbit coupling and the like.



In QFT, stuff starts to get a bit/lot confusing. First, angular momentum operator is really used to label states in the irreducible unitary representation of the Poincare group, which is written as $left|p,sigmaright>$ (or $Psi_{p,s}$ following Weinberg); it is an operator in the sense that it appears as a representation of rotation element of the group. Wigner's classification then gives us that we also label states with two quantum numbers: we sometimes also write this as $left|m,jright>$, where $m$ is the rest mass and $j$ is the spin. Nowhere in any of these constructions do I see the conventional orbital angular momentum (in the text by Schwartz Chapter 11, for example, he made a comment when there is no angular momentum, so he got the "easy" case of $vec{J}=vec{S}$.) In standard QM, we clearly distinguish two versions of angular momenta and their eigenvalues.



Question: does orbital angular momentum make sense in QFT and how does it arise if it does? Should I think of $vec L$ as furnishing some kind of tensor product representation so that I should be labelling states with $left|m,j,sright>$ or something? Another possibility is that one simply does not work with addition of angular momenta in QFT, especially in free theory, but I would like to have an explanation on why this should or should not be the case. I may have understood something really basic but I can't fish out exactly what.



Note: I believe the usual thing about spin-orbit coupling should not work, because that's QM (think about hydrogen atom) and strictly speaking QFT, unless we do something more (2-particle states?). In those cases, the operator $vec{L}$ even comes from $vec{r}timesvec{p}$ which does not appear naturally in QFT (what's $vec{r}$ in QFT?). I don't think I should go and define $vec{L}:=vec{r}timesvec{hat{pi}}$, where $hatpi$ is the conjugate momentum of the field.







quantum-field-theory angular-momentum quantum-spin spinors






share|cite|improve this question















share|cite|improve this question













share|cite|improve this question




share|cite|improve this question








edited 2 hours ago







Everiana

















asked 8 hours ago









EverianaEveriana

5252 silver badges14 bronze badges




5252 silver badges14 bronze badges












  • $begingroup$
    Why is "that not QFT?" Angular momentum, both spin and orbital, is strongly related to rotational symmetry. (Paging E. Noether. Will E. Noether please pick up a white house phone.) You still have symmetry in QFT. You still get the same reps of the rotation group.
    $endgroup$
    – puppetsock
    8 hours ago


















  • $begingroup$
    Why is "that not QFT?" Angular momentum, both spin and orbital, is strongly related to rotational symmetry. (Paging E. Noether. Will E. Noether please pick up a white house phone.) You still have symmetry in QFT. You still get the same reps of the rotation group.
    $endgroup$
    – puppetsock
    8 hours ago
















$begingroup$
Why is "that not QFT?" Angular momentum, both spin and orbital, is strongly related to rotational symmetry. (Paging E. Noether. Will E. Noether please pick up a white house phone.) You still have symmetry in QFT. You still get the same reps of the rotation group.
$endgroup$
– puppetsock
8 hours ago




$begingroup$
Why is "that not QFT?" Angular momentum, both spin and orbital, is strongly related to rotational symmetry. (Paging E. Noether. Will E. Noether please pick up a white house phone.) You still have symmetry in QFT. You still get the same reps of the rotation group.
$endgroup$
– puppetsock
8 hours ago










3 Answers
3






active

oldest

votes


















2












$begingroup$

Since you mention Weinberg, have a look at section 7.4, especially (7.4.10). Under a homogeneous Lorentz transformation generated by the anti-symmetric tensor $omega^{munu}$, a field transforms like
$$Psi^ell mapsto Psi^ell + (mathcal{J}_{munu})^ell{}_m omega^{munu} Psi^m$$
i.e. with the representative of $omega^{munu}$ in whatever representation $Psi$ belongs to. Then according to Noether's theorem
$$ 0 = partial^kappa left[frac{i}{2} frac{partial mathcal{L}}{partial (partial^kappa Psi^ell)} (mathcal{J}_{munu})^ell{}_m Psi^m right] - frac{1}{2}(T_{munu} - T_{numu}) tag{7.4.10}$$
where $T_{munu}$ is the Noether current of translations, i.e., the canonical stress-energy tensor.



Now, $T_{munu}$ is itself conserved, $partial^mu T_{munu} = 0$, so
$$partial^kappa (x_mu T_{kappanu} - x_nu T_{kappa mu}) = T_{munu} - T_{numu}$$ and we can write (7.4.10) as
$$
0 = partial^kappa left[
frac{i}{2} frac{partial mathcal{L}}{partial (partial^kappa Psi^ell)} (mathcal{J}_{munu})^ell{}_m Psi^m
- frac{1}{2}(x_mu T_{kappanu} - x_nu T_{kappa mu})
right ]
$$

and this is quite clearly expressing that spin plus orbital angular momentum is conserved.



Read further in Weinberg 7.4 and see also the second section of the Wikipedia page on the Belinfante-Rosenfeld tensor.






share|cite|improve this answer











$endgroup$





















    1












    $begingroup$

    Orbital angular momentum has to do with position. It is the 'ordinary' angular momentum $rtimes p$ we learned about in Physics 101 whereas spin is an extra internal angular momentum that doesn't have to do with position. Now there is one extra label you forgot on your Wigner one particle state basis. That is the momentum $p$.



    When we rotate the state $p$ rotates too, so the angular momentum operator should have a part related to the momentum operator. If we have a wavepacket superposition this extra piece will act like the 'ordinary' orbital angular momentum part.



    This is accounted for automatically if you find the angular momentum operator in terms of fields via the Lagrangian. The angular momentum density will look something like $x^lambda T^{munu}-x^mu T^{lambdanu}$ where $T$ is the energy momentum tensor, plus an aditional 'spin' part if the fundamental field(s) in your Lagrangian have non-trivial rotation properties.






    share|cite|improve this answer









    $endgroup$





















      1












      $begingroup$

      If you think about Wigner's classification, then one-particle states are irreducible representations of the Poincare group, as you said. To label these representations you need the Casimir invariants of Poincare - the particle's mass and the little group Casimir, which for massive particles corresponds to the symmetries in is its rest frame, the SO(3) rotation group. That's how you get spin and that's all the angular momentum you have in one-particle states. Orbital angular momentum requires the concept of distance, which for 1-particle states does not exist.



      However, if you consider two-particle states you can just take it as a tensor product between 2 one-particle states with 2 different spin labels. OR, you can take it as a single state with definite energy-momentum and total angular momentum label. This angular momentum is just the Clebsch-Gordon sum of both spins and the orbital angular momentum between the particles. How you move from one description to the other is by going to the center of mass of the two particles. Here you have to specify the direction of the relative linear momentum on the 2-sphere, this two-angle dependence can be expanded into spherical harmonics, eigenfunctions of angular momentum.



      As a concrete example, consider two spin-0 bosons. This 2-particle state depends on 6 variables, 2x linear momentum vectors. If you go to CM frame this state depends on the total linear momentum and the relative momentum. The absolute value of the relative momentum can be put into the total Energy, and you're only missing the orientation of the relative momentum. The 2 angle dependence can be traded for (l,m) by a spherical harmonic decomposition. If the particles have internal spin then you can sum these spins using Clebsch-Gordon coefficients, to get the usual total angular momentum 'j'. You can then check that these 'j' states are indeed eigeinstates of the 2-particle representation of the rotation generators of the Poincare group.






      share|cite|improve this answer








      New contributor



      Miguel Correia is a new contributor to this site. Take care in asking for clarification, commenting, and answering.
      Check out our Code of Conduct.





      $endgroup$
















        Your Answer








        StackExchange.ready(function() {
        var channelOptions = {
        tags: "".split(" "),
        id: "151"
        };
        initTagRenderer("".split(" "), "".split(" "), channelOptions);

        StackExchange.using("externalEditor", function() {
        // Have to fire editor after snippets, if snippets enabled
        if (StackExchange.settings.snippets.snippetsEnabled) {
        StackExchange.using("snippets", function() {
        createEditor();
        });
        }
        else {
        createEditor();
        }
        });

        function createEditor() {
        StackExchange.prepareEditor({
        heartbeatType: 'answer',
        autoActivateHeartbeat: false,
        convertImagesToLinks: false,
        noModals: true,
        showLowRepImageUploadWarning: true,
        reputationToPostImages: null,
        bindNavPrevention: true,
        postfix: "",
        imageUploader: {
        brandingHtml: "Powered by u003ca class="icon-imgur-white" href="https://imgur.com/"u003eu003c/au003e",
        contentPolicyHtml: "User contributions licensed under u003ca href="https://creativecommons.org/licenses/by-sa/3.0/"u003ecc by-sa 3.0 with attribution requiredu003c/au003e u003ca href="https://stackoverflow.com/legal/content-policy"u003e(content policy)u003c/au003e",
        allowUrls: true
        },
        noCode: true, onDemand: true,
        discardSelector: ".discard-answer"
        ,immediatelyShowMarkdownHelp:true
        });


        }
        });














        draft saved

        draft discarded


















        StackExchange.ready(
        function () {
        StackExchange.openid.initPostLogin('.new-post-login', 'https%3a%2f%2fphysics.stackexchange.com%2fquestions%2f493792%2fspin-vs-orbital-angular-momenta-in-qft%23new-answer', 'question_page');
        }
        );

        Post as a guest















        Required, but never shown

























        3 Answers
        3






        active

        oldest

        votes








        3 Answers
        3






        active

        oldest

        votes









        active

        oldest

        votes






        active

        oldest

        votes









        2












        $begingroup$

        Since you mention Weinberg, have a look at section 7.4, especially (7.4.10). Under a homogeneous Lorentz transformation generated by the anti-symmetric tensor $omega^{munu}$, a field transforms like
        $$Psi^ell mapsto Psi^ell + (mathcal{J}_{munu})^ell{}_m omega^{munu} Psi^m$$
        i.e. with the representative of $omega^{munu}$ in whatever representation $Psi$ belongs to. Then according to Noether's theorem
        $$ 0 = partial^kappa left[frac{i}{2} frac{partial mathcal{L}}{partial (partial^kappa Psi^ell)} (mathcal{J}_{munu})^ell{}_m Psi^m right] - frac{1}{2}(T_{munu} - T_{numu}) tag{7.4.10}$$
        where $T_{munu}$ is the Noether current of translations, i.e., the canonical stress-energy tensor.



        Now, $T_{munu}$ is itself conserved, $partial^mu T_{munu} = 0$, so
        $$partial^kappa (x_mu T_{kappanu} - x_nu T_{kappa mu}) = T_{munu} - T_{numu}$$ and we can write (7.4.10) as
        $$
        0 = partial^kappa left[
        frac{i}{2} frac{partial mathcal{L}}{partial (partial^kappa Psi^ell)} (mathcal{J}_{munu})^ell{}_m Psi^m
        - frac{1}{2}(x_mu T_{kappanu} - x_nu T_{kappa mu})
        right ]
        $$

        and this is quite clearly expressing that spin plus orbital angular momentum is conserved.



        Read further in Weinberg 7.4 and see also the second section of the Wikipedia page on the Belinfante-Rosenfeld tensor.






        share|cite|improve this answer











        $endgroup$


















          2












          $begingroup$

          Since you mention Weinberg, have a look at section 7.4, especially (7.4.10). Under a homogeneous Lorentz transformation generated by the anti-symmetric tensor $omega^{munu}$, a field transforms like
          $$Psi^ell mapsto Psi^ell + (mathcal{J}_{munu})^ell{}_m omega^{munu} Psi^m$$
          i.e. with the representative of $omega^{munu}$ in whatever representation $Psi$ belongs to. Then according to Noether's theorem
          $$ 0 = partial^kappa left[frac{i}{2} frac{partial mathcal{L}}{partial (partial^kappa Psi^ell)} (mathcal{J}_{munu})^ell{}_m Psi^m right] - frac{1}{2}(T_{munu} - T_{numu}) tag{7.4.10}$$
          where $T_{munu}$ is the Noether current of translations, i.e., the canonical stress-energy tensor.



          Now, $T_{munu}$ is itself conserved, $partial^mu T_{munu} = 0$, so
          $$partial^kappa (x_mu T_{kappanu} - x_nu T_{kappa mu}) = T_{munu} - T_{numu}$$ and we can write (7.4.10) as
          $$
          0 = partial^kappa left[
          frac{i}{2} frac{partial mathcal{L}}{partial (partial^kappa Psi^ell)} (mathcal{J}_{munu})^ell{}_m Psi^m
          - frac{1}{2}(x_mu T_{kappanu} - x_nu T_{kappa mu})
          right ]
          $$

          and this is quite clearly expressing that spin plus orbital angular momentum is conserved.



          Read further in Weinberg 7.4 and see also the second section of the Wikipedia page on the Belinfante-Rosenfeld tensor.






          share|cite|improve this answer











          $endgroup$
















            2












            2








            2





            $begingroup$

            Since you mention Weinberg, have a look at section 7.4, especially (7.4.10). Under a homogeneous Lorentz transformation generated by the anti-symmetric tensor $omega^{munu}$, a field transforms like
            $$Psi^ell mapsto Psi^ell + (mathcal{J}_{munu})^ell{}_m omega^{munu} Psi^m$$
            i.e. with the representative of $omega^{munu}$ in whatever representation $Psi$ belongs to. Then according to Noether's theorem
            $$ 0 = partial^kappa left[frac{i}{2} frac{partial mathcal{L}}{partial (partial^kappa Psi^ell)} (mathcal{J}_{munu})^ell{}_m Psi^m right] - frac{1}{2}(T_{munu} - T_{numu}) tag{7.4.10}$$
            where $T_{munu}$ is the Noether current of translations, i.e., the canonical stress-energy tensor.



            Now, $T_{munu}$ is itself conserved, $partial^mu T_{munu} = 0$, so
            $$partial^kappa (x_mu T_{kappanu} - x_nu T_{kappa mu}) = T_{munu} - T_{numu}$$ and we can write (7.4.10) as
            $$
            0 = partial^kappa left[
            frac{i}{2} frac{partial mathcal{L}}{partial (partial^kappa Psi^ell)} (mathcal{J}_{munu})^ell{}_m Psi^m
            - frac{1}{2}(x_mu T_{kappanu} - x_nu T_{kappa mu})
            right ]
            $$

            and this is quite clearly expressing that spin plus orbital angular momentum is conserved.



            Read further in Weinberg 7.4 and see also the second section of the Wikipedia page on the Belinfante-Rosenfeld tensor.






            share|cite|improve this answer











            $endgroup$



            Since you mention Weinberg, have a look at section 7.4, especially (7.4.10). Under a homogeneous Lorentz transformation generated by the anti-symmetric tensor $omega^{munu}$, a field transforms like
            $$Psi^ell mapsto Psi^ell + (mathcal{J}_{munu})^ell{}_m omega^{munu} Psi^m$$
            i.e. with the representative of $omega^{munu}$ in whatever representation $Psi$ belongs to. Then according to Noether's theorem
            $$ 0 = partial^kappa left[frac{i}{2} frac{partial mathcal{L}}{partial (partial^kappa Psi^ell)} (mathcal{J}_{munu})^ell{}_m Psi^m right] - frac{1}{2}(T_{munu} - T_{numu}) tag{7.4.10}$$
            where $T_{munu}$ is the Noether current of translations, i.e., the canonical stress-energy tensor.



            Now, $T_{munu}$ is itself conserved, $partial^mu T_{munu} = 0$, so
            $$partial^kappa (x_mu T_{kappanu} - x_nu T_{kappa mu}) = T_{munu} - T_{numu}$$ and we can write (7.4.10) as
            $$
            0 = partial^kappa left[
            frac{i}{2} frac{partial mathcal{L}}{partial (partial^kappa Psi^ell)} (mathcal{J}_{munu})^ell{}_m Psi^m
            - frac{1}{2}(x_mu T_{kappanu} - x_nu T_{kappa mu})
            right ]
            $$

            and this is quite clearly expressing that spin plus orbital angular momentum is conserved.



            Read further in Weinberg 7.4 and see also the second section of the Wikipedia page on the Belinfante-Rosenfeld tensor.







            share|cite|improve this answer














            share|cite|improve this answer



            share|cite|improve this answer








            edited 4 hours ago

























            answered 4 hours ago









            Robin EkmanRobin Ekman

            12.2k1 gold badge22 silver badges48 bronze badges




            12.2k1 gold badge22 silver badges48 bronze badges

























                1












                $begingroup$

                Orbital angular momentum has to do with position. It is the 'ordinary' angular momentum $rtimes p$ we learned about in Physics 101 whereas spin is an extra internal angular momentum that doesn't have to do with position. Now there is one extra label you forgot on your Wigner one particle state basis. That is the momentum $p$.



                When we rotate the state $p$ rotates too, so the angular momentum operator should have a part related to the momentum operator. If we have a wavepacket superposition this extra piece will act like the 'ordinary' orbital angular momentum part.



                This is accounted for automatically if you find the angular momentum operator in terms of fields via the Lagrangian. The angular momentum density will look something like $x^lambda T^{munu}-x^mu T^{lambdanu}$ where $T$ is the energy momentum tensor, plus an aditional 'spin' part if the fundamental field(s) in your Lagrangian have non-trivial rotation properties.






                share|cite|improve this answer









                $endgroup$


















                  1












                  $begingroup$

                  Orbital angular momentum has to do with position. It is the 'ordinary' angular momentum $rtimes p$ we learned about in Physics 101 whereas spin is an extra internal angular momentum that doesn't have to do with position. Now there is one extra label you forgot on your Wigner one particle state basis. That is the momentum $p$.



                  When we rotate the state $p$ rotates too, so the angular momentum operator should have a part related to the momentum operator. If we have a wavepacket superposition this extra piece will act like the 'ordinary' orbital angular momentum part.



                  This is accounted for automatically if you find the angular momentum operator in terms of fields via the Lagrangian. The angular momentum density will look something like $x^lambda T^{munu}-x^mu T^{lambdanu}$ where $T$ is the energy momentum tensor, plus an aditional 'spin' part if the fundamental field(s) in your Lagrangian have non-trivial rotation properties.






                  share|cite|improve this answer









                  $endgroup$
















                    1












                    1








                    1





                    $begingroup$

                    Orbital angular momentum has to do with position. It is the 'ordinary' angular momentum $rtimes p$ we learned about in Physics 101 whereas spin is an extra internal angular momentum that doesn't have to do with position. Now there is one extra label you forgot on your Wigner one particle state basis. That is the momentum $p$.



                    When we rotate the state $p$ rotates too, so the angular momentum operator should have a part related to the momentum operator. If we have a wavepacket superposition this extra piece will act like the 'ordinary' orbital angular momentum part.



                    This is accounted for automatically if you find the angular momentum operator in terms of fields via the Lagrangian. The angular momentum density will look something like $x^lambda T^{munu}-x^mu T^{lambdanu}$ where $T$ is the energy momentum tensor, plus an aditional 'spin' part if the fundamental field(s) in your Lagrangian have non-trivial rotation properties.






                    share|cite|improve this answer









                    $endgroup$



                    Orbital angular momentum has to do with position. It is the 'ordinary' angular momentum $rtimes p$ we learned about in Physics 101 whereas spin is an extra internal angular momentum that doesn't have to do with position. Now there is one extra label you forgot on your Wigner one particle state basis. That is the momentum $p$.



                    When we rotate the state $p$ rotates too, so the angular momentum operator should have a part related to the momentum operator. If we have a wavepacket superposition this extra piece will act like the 'ordinary' orbital angular momentum part.



                    This is accounted for automatically if you find the angular momentum operator in terms of fields via the Lagrangian. The angular momentum density will look something like $x^lambda T^{munu}-x^mu T^{lambdanu}$ where $T$ is the energy momentum tensor, plus an aditional 'spin' part if the fundamental field(s) in your Lagrangian have non-trivial rotation properties.







                    share|cite|improve this answer












                    share|cite|improve this answer



                    share|cite|improve this answer










                    answered 6 hours ago









                    octonionoctonion

                    4,3168 silver badges18 bronze badges




                    4,3168 silver badges18 bronze badges























                        1












                        $begingroup$

                        If you think about Wigner's classification, then one-particle states are irreducible representations of the Poincare group, as you said. To label these representations you need the Casimir invariants of Poincare - the particle's mass and the little group Casimir, which for massive particles corresponds to the symmetries in is its rest frame, the SO(3) rotation group. That's how you get spin and that's all the angular momentum you have in one-particle states. Orbital angular momentum requires the concept of distance, which for 1-particle states does not exist.



                        However, if you consider two-particle states you can just take it as a tensor product between 2 one-particle states with 2 different spin labels. OR, you can take it as a single state with definite energy-momentum and total angular momentum label. This angular momentum is just the Clebsch-Gordon sum of both spins and the orbital angular momentum between the particles. How you move from one description to the other is by going to the center of mass of the two particles. Here you have to specify the direction of the relative linear momentum on the 2-sphere, this two-angle dependence can be expanded into spherical harmonics, eigenfunctions of angular momentum.



                        As a concrete example, consider two spin-0 bosons. This 2-particle state depends on 6 variables, 2x linear momentum vectors. If you go to CM frame this state depends on the total linear momentum and the relative momentum. The absolute value of the relative momentum can be put into the total Energy, and you're only missing the orientation of the relative momentum. The 2 angle dependence can be traded for (l,m) by a spherical harmonic decomposition. If the particles have internal spin then you can sum these spins using Clebsch-Gordon coefficients, to get the usual total angular momentum 'j'. You can then check that these 'j' states are indeed eigeinstates of the 2-particle representation of the rotation generators of the Poincare group.






                        share|cite|improve this answer








                        New contributor



                        Miguel Correia is a new contributor to this site. Take care in asking for clarification, commenting, and answering.
                        Check out our Code of Conduct.





                        $endgroup$


















                          1












                          $begingroup$

                          If you think about Wigner's classification, then one-particle states are irreducible representations of the Poincare group, as you said. To label these representations you need the Casimir invariants of Poincare - the particle's mass and the little group Casimir, which for massive particles corresponds to the symmetries in is its rest frame, the SO(3) rotation group. That's how you get spin and that's all the angular momentum you have in one-particle states. Orbital angular momentum requires the concept of distance, which for 1-particle states does not exist.



                          However, if you consider two-particle states you can just take it as a tensor product between 2 one-particle states with 2 different spin labels. OR, you can take it as a single state with definite energy-momentum and total angular momentum label. This angular momentum is just the Clebsch-Gordon sum of both spins and the orbital angular momentum between the particles. How you move from one description to the other is by going to the center of mass of the two particles. Here you have to specify the direction of the relative linear momentum on the 2-sphere, this two-angle dependence can be expanded into spherical harmonics, eigenfunctions of angular momentum.



                          As a concrete example, consider two spin-0 bosons. This 2-particle state depends on 6 variables, 2x linear momentum vectors. If you go to CM frame this state depends on the total linear momentum and the relative momentum. The absolute value of the relative momentum can be put into the total Energy, and you're only missing the orientation of the relative momentum. The 2 angle dependence can be traded for (l,m) by a spherical harmonic decomposition. If the particles have internal spin then you can sum these spins using Clebsch-Gordon coefficients, to get the usual total angular momentum 'j'. You can then check that these 'j' states are indeed eigeinstates of the 2-particle representation of the rotation generators of the Poincare group.






                          share|cite|improve this answer








                          New contributor



                          Miguel Correia is a new contributor to this site. Take care in asking for clarification, commenting, and answering.
                          Check out our Code of Conduct.





                          $endgroup$
















                            1












                            1








                            1





                            $begingroup$

                            If you think about Wigner's classification, then one-particle states are irreducible representations of the Poincare group, as you said. To label these representations you need the Casimir invariants of Poincare - the particle's mass and the little group Casimir, which for massive particles corresponds to the symmetries in is its rest frame, the SO(3) rotation group. That's how you get spin and that's all the angular momentum you have in one-particle states. Orbital angular momentum requires the concept of distance, which for 1-particle states does not exist.



                            However, if you consider two-particle states you can just take it as a tensor product between 2 one-particle states with 2 different spin labels. OR, you can take it as a single state with definite energy-momentum and total angular momentum label. This angular momentum is just the Clebsch-Gordon sum of both spins and the orbital angular momentum between the particles. How you move from one description to the other is by going to the center of mass of the two particles. Here you have to specify the direction of the relative linear momentum on the 2-sphere, this two-angle dependence can be expanded into spherical harmonics, eigenfunctions of angular momentum.



                            As a concrete example, consider two spin-0 bosons. This 2-particle state depends on 6 variables, 2x linear momentum vectors. If you go to CM frame this state depends on the total linear momentum and the relative momentum. The absolute value of the relative momentum can be put into the total Energy, and you're only missing the orientation of the relative momentum. The 2 angle dependence can be traded for (l,m) by a spherical harmonic decomposition. If the particles have internal spin then you can sum these spins using Clebsch-Gordon coefficients, to get the usual total angular momentum 'j'. You can then check that these 'j' states are indeed eigeinstates of the 2-particle representation of the rotation generators of the Poincare group.






                            share|cite|improve this answer








                            New contributor



                            Miguel Correia is a new contributor to this site. Take care in asking for clarification, commenting, and answering.
                            Check out our Code of Conduct.





                            $endgroup$



                            If you think about Wigner's classification, then one-particle states are irreducible representations of the Poincare group, as you said. To label these representations you need the Casimir invariants of Poincare - the particle's mass and the little group Casimir, which for massive particles corresponds to the symmetries in is its rest frame, the SO(3) rotation group. That's how you get spin and that's all the angular momentum you have in one-particle states. Orbital angular momentum requires the concept of distance, which for 1-particle states does not exist.



                            However, if you consider two-particle states you can just take it as a tensor product between 2 one-particle states with 2 different spin labels. OR, you can take it as a single state with definite energy-momentum and total angular momentum label. This angular momentum is just the Clebsch-Gordon sum of both spins and the orbital angular momentum between the particles. How you move from one description to the other is by going to the center of mass of the two particles. Here you have to specify the direction of the relative linear momentum on the 2-sphere, this two-angle dependence can be expanded into spherical harmonics, eigenfunctions of angular momentum.



                            As a concrete example, consider two spin-0 bosons. This 2-particle state depends on 6 variables, 2x linear momentum vectors. If you go to CM frame this state depends on the total linear momentum and the relative momentum. The absolute value of the relative momentum can be put into the total Energy, and you're only missing the orientation of the relative momentum. The 2 angle dependence can be traded for (l,m) by a spherical harmonic decomposition. If the particles have internal spin then you can sum these spins using Clebsch-Gordon coefficients, to get the usual total angular momentum 'j'. You can then check that these 'j' states are indeed eigeinstates of the 2-particle representation of the rotation generators of the Poincare group.







                            share|cite|improve this answer








                            New contributor



                            Miguel Correia is a new contributor to this site. Take care in asking for clarification, commenting, and answering.
                            Check out our Code of Conduct.








                            share|cite|improve this answer



                            share|cite|improve this answer






                            New contributor



                            Miguel Correia is a new contributor to this site. Take care in asking for clarification, commenting, and answering.
                            Check out our Code of Conduct.








                            answered 4 hours ago









                            Miguel CorreiaMiguel Correia

                            112 bronze badges




                            112 bronze badges




                            New contributor



                            Miguel Correia is a new contributor to this site. Take care in asking for clarification, commenting, and answering.
                            Check out our Code of Conduct.




                            New contributor




                            Miguel Correia is a new contributor to this site. Take care in asking for clarification, commenting, and answering.
                            Check out our Code of Conduct.
































                                draft saved

                                draft discarded




















































                                Thanks for contributing an answer to Physics Stack Exchange!


                                • Please be sure to answer the question. Provide details and share your research!

                                But avoid



                                • Asking for help, clarification, or responding to other answers.

                                • Making statements based on opinion; back them up with references or personal experience.


                                Use MathJax to format equations. MathJax reference.


                                To learn more, see our tips on writing great answers.




                                draft saved


                                draft discarded














                                StackExchange.ready(
                                function () {
                                StackExchange.openid.initPostLogin('.new-post-login', 'https%3a%2f%2fphysics.stackexchange.com%2fquestions%2f493792%2fspin-vs-orbital-angular-momenta-in-qft%23new-answer', 'question_page');
                                }
                                );

                                Post as a guest















                                Required, but never shown





















































                                Required, but never shown














                                Required, but never shown












                                Required, but never shown







                                Required, but never shown

































                                Required, but never shown














                                Required, but never shown












                                Required, but never shown







                                Required, but never shown







                                Popular posts from this blog

                                Taj Mahal Inhaltsverzeichnis Aufbau | Geschichte | 350-Jahr-Feier | Heutige Bedeutung | Siehe auch |...

                                Baia Sprie Cuprins Etimologie | Istorie | Demografie | Politică și administrație | Arii naturale...

                                Ciclooctatetraenă Vezi și | Bibliografie | Meniu de navigare637866text4148569-500570979m